Find the sum: $sum_{n=2}^infty frac{1}{n^2-1}$











up vote
3
down vote

favorite













Evaluate : $$sum_{n=2}^infty frac{1}{n^2-1}$$




I've tried to rewrite the questions as $$sum _{n=2}^{infty ::}left(-frac{1}{2left(n+1right)}+frac{1}{2left(n-1right)}right)$$ but still couldnt't get any answer as when I substitute numbers $(n)$ into the $Sn left(-frac{1}{6}+frac{1}{2}-frac{1}{8}+frac{1}{4}-frac{1}{10}right)$ , $Sn$ just keeps going on and on. So how do I solve this problem and what does this series converge to?










share|cite|improve this question
























  • What does mean $[.]$ ?
    – Nosrati
    Nov 23 at 3:03










  • That was me @Nosrati, its just a square bracket to encase the argument of the sum.
    – Rhys Hughes
    Nov 23 at 3:05










  • Keyword: telescoping.
    – Cheerful Parsnip
    Nov 23 at 3:50










  • Is my answer okay?
    – Akash Roy
    Nov 23 at 3:51















up vote
3
down vote

favorite













Evaluate : $$sum_{n=2}^infty frac{1}{n^2-1}$$




I've tried to rewrite the questions as $$sum _{n=2}^{infty ::}left(-frac{1}{2left(n+1right)}+frac{1}{2left(n-1right)}right)$$ but still couldnt't get any answer as when I substitute numbers $(n)$ into the $Sn left(-frac{1}{6}+frac{1}{2}-frac{1}{8}+frac{1}{4}-frac{1}{10}right)$ , $Sn$ just keeps going on and on. So how do I solve this problem and what does this series converge to?










share|cite|improve this question
























  • What does mean $[.]$ ?
    – Nosrati
    Nov 23 at 3:03










  • That was me @Nosrati, its just a square bracket to encase the argument of the sum.
    – Rhys Hughes
    Nov 23 at 3:05










  • Keyword: telescoping.
    – Cheerful Parsnip
    Nov 23 at 3:50










  • Is my answer okay?
    – Akash Roy
    Nov 23 at 3:51













up vote
3
down vote

favorite









up vote
3
down vote

favorite












Evaluate : $$sum_{n=2}^infty frac{1}{n^2-1}$$




I've tried to rewrite the questions as $$sum _{n=2}^{infty ::}left(-frac{1}{2left(n+1right)}+frac{1}{2left(n-1right)}right)$$ but still couldnt't get any answer as when I substitute numbers $(n)$ into the $Sn left(-frac{1}{6}+frac{1}{2}-frac{1}{8}+frac{1}{4}-frac{1}{10}right)$ , $Sn$ just keeps going on and on. So how do I solve this problem and what does this series converge to?










share|cite|improve this question
















Evaluate : $$sum_{n=2}^infty frac{1}{n^2-1}$$




I've tried to rewrite the questions as $$sum _{n=2}^{infty ::}left(-frac{1}{2left(n+1right)}+frac{1}{2left(n-1right)}right)$$ but still couldnt't get any answer as when I substitute numbers $(n)$ into the $Sn left(-frac{1}{6}+frac{1}{2}-frac{1}{8}+frac{1}{4}-frac{1}{10}right)$ , $Sn$ just keeps going on and on. So how do I solve this problem and what does this series converge to?







sequences-and-series limits convergence






share|cite|improve this question















share|cite|improve this question













share|cite|improve this question




share|cite|improve this question








edited Nov 23 at 3:31









Chinnapparaj R

4,9951825




4,9951825










asked Nov 23 at 2:56









Fourth

514




514












  • What does mean $[.]$ ?
    – Nosrati
    Nov 23 at 3:03










  • That was me @Nosrati, its just a square bracket to encase the argument of the sum.
    – Rhys Hughes
    Nov 23 at 3:05










  • Keyword: telescoping.
    – Cheerful Parsnip
    Nov 23 at 3:50










  • Is my answer okay?
    – Akash Roy
    Nov 23 at 3:51


















  • What does mean $[.]$ ?
    – Nosrati
    Nov 23 at 3:03










  • That was me @Nosrati, its just a square bracket to encase the argument of the sum.
    – Rhys Hughes
    Nov 23 at 3:05










  • Keyword: telescoping.
    – Cheerful Parsnip
    Nov 23 at 3:50










  • Is my answer okay?
    – Akash Roy
    Nov 23 at 3:51
















What does mean $[.]$ ?
– Nosrati
Nov 23 at 3:03




What does mean $[.]$ ?
– Nosrati
Nov 23 at 3:03












That was me @Nosrati, its just a square bracket to encase the argument of the sum.
– Rhys Hughes
Nov 23 at 3:05




That was me @Nosrati, its just a square bracket to encase the argument of the sum.
– Rhys Hughes
Nov 23 at 3:05












Keyword: telescoping.
– Cheerful Parsnip
Nov 23 at 3:50




Keyword: telescoping.
– Cheerful Parsnip
Nov 23 at 3:50












Is my answer okay?
– Akash Roy
Nov 23 at 3:51




Is my answer okay?
– Akash Roy
Nov 23 at 3:51










4 Answers
4






active

oldest

votes

















up vote
3
down vote













$$frac{1}{n^2-1}=frac{1}{(n-1)(n+1)}=frac{1}{2(n-1)}-frac{1}{2(n+1)}$$



Taking $frac{1}{2}$ common ,



$$sum_{n=2}^{infty}frac{1}{n^2-1}=frac12sum_{n=2}^{infty}left(frac{1}{n-1}-frac{1}{n+1}right)$$
Write the first few terms of the series as :



$frac{1}{2}( 1 - frac{1}{3} +frac{1}{2} - frac{1}{4} + frac{1}{3} - frac{1}{5} + frac{1}{4} - cdot cdot cdot$



You can see that except $1$ and $frac{1}{2}$ every terms get cancelled out and the $n$-th term tends to zero. So the result inside the bracket is $frac{3}{2}$ . But there is a $frac{1}{2}$ outside the bracket which needs to be multiplied. So the answer is $frac{3}{4}$.






share|cite|improve this answer





















  • Yes, this looks ok to me.
    – Jimmy R.
    Nov 23 at 4:27


















up vote
2
down vote













As you correctly have: $$frac{1}{n^2-1}=frac{1}{(n-1)(n+1)}=frac{1}{2(n-1)}-frac{1}{2(n+1)}$$ Now, observe that $$sum_{n=2}^{infty}frac{1}{n-1}=sum_{n=1}^{infty}frac{1}{n} qquad text{and}qquadsum_{n=2}^{infty}frac{1}{n+1}=sum_{n=3}^{infty}frac{1}{n}$$ Hence
$$sum_{n=2}^{infty}frac{1}{n^2-1}=frac12sum_{n=2}^{infty}left(frac{1}{n-1}-frac{1}{n+1}right)=frac12left(sum_{n=1}^{infty}frac1n-sum_{n=3}^{infty}frac1nright)=frac12left(frac11+frac12right)=frac34$$






share|cite|improve this answer



















  • 5




    If you consider partial sums, you can avoid writing out the divergent harmonic series which makes me quite comfortable :)
    – Szeto
    Nov 23 at 3:35






  • 3




    @Szeto: +1 to that. I can prove many things if I'm allowed to use expressions like $sum_{n=1}^{infty}frac{1}{n}$....
    – mjqxxxx
    Nov 23 at 3:39










  • Is my answer okay please check once
    – Akash Roy
    Nov 23 at 3:56


















up vote
1
down vote













More generally,
if
$s_m(n)
=sum_{k=m+1}^n frac{1}{k^2-m^2}
$

then,
if $n > 3m$,



$begin{array}\
s_m(n)
&=sum_{k=m+1}^n frac{1}{k^2-m^2}\
&=sum_{k=m+1}^n frac1{2m}(frac{1}{k-m}-frac1{k+m})\
&=frac1{2m}sum_{k=m+1}^n frac{1}{k-m}-frac1{2m}sum_{k=m+1}^nfrac1{k+m}\
&=frac1{2m}(sum_{k=1}^{2m} frac{1}{k}+sum_{k=2m+1}^{n-m} frac{1}{k})-(frac1{2m}sum_{k=2m+1}^{n-m}frac1{k}+frac1{2m}sum_{k=n-m+1}^{n+m}frac1{k})\
&=frac1{2m}sum_{k=1}^{2m} frac{1}{k}-frac1{2m}sum_{k=n-m+1}^{n+m}frac1{k}\
end{array}
$



so



$begin{array}\
s_m(n)-frac1{2m}sum_{k=1}^{2m} frac{1}{k}
&=-frac1{2m}sum_{k=n-m+1}^{n+m}frac1{k}\
text{so that}\
|s_m(n)-frac1{2m}sum_{k=1}^{2m} frac{1}{k}|
&=frac1{2m}|sum_{k=n-m+1}^{n+m}frac1{k}|\
&lefrac1{2m}|sum_{k=n-m+1}^{n+m}frac1{n-m+1}|\
&=frac1{2m}|frac{2m}{n-m+1}|\
&=frac{1}{n-m+1}\
&to 0
quadtext{ as } n to infty\
end{array}
$



Therefore
$lim_{n to infty} s_m(n)
=frac1{2m}sum_{k=1}^{2m} frac{1}{k}
$
.



For $m=1$
the sum is
$frac1{2}(frac1{1}+frac1{2})
=frac34
$
.






share|cite|improve this answer




























    up vote
    0
    down vote













    HINT:



    $$frac{1}{2[(n+2)-1]}-frac{1}{2[n+1]}=0$$






    share|cite|improve this answer





















    • Is my answer okay? Please check
      – Akash Roy
      Nov 23 at 3:56











    Your Answer





    StackExchange.ifUsing("editor", function () {
    return StackExchange.using("mathjaxEditing", function () {
    StackExchange.MarkdownEditor.creationCallbacks.add(function (editor, postfix) {
    StackExchange.mathjaxEditing.prepareWmdForMathJax(editor, postfix, [["$", "$"], ["\\(","\\)"]]);
    });
    });
    }, "mathjax-editing");

    StackExchange.ready(function() {
    var channelOptions = {
    tags: "".split(" "),
    id: "69"
    };
    initTagRenderer("".split(" "), "".split(" "), channelOptions);

    StackExchange.using("externalEditor", function() {
    // Have to fire editor after snippets, if snippets enabled
    if (StackExchange.settings.snippets.snippetsEnabled) {
    StackExchange.using("snippets", function() {
    createEditor();
    });
    }
    else {
    createEditor();
    }
    });

    function createEditor() {
    StackExchange.prepareEditor({
    heartbeatType: 'answer',
    convertImagesToLinks: true,
    noModals: true,
    showLowRepImageUploadWarning: true,
    reputationToPostImages: 10,
    bindNavPrevention: true,
    postfix: "",
    imageUploader: {
    brandingHtml: "Powered by u003ca class="icon-imgur-white" href="https://imgur.com/"u003eu003c/au003e",
    contentPolicyHtml: "User contributions licensed under u003ca href="https://creativecommons.org/licenses/by-sa/3.0/"u003ecc by-sa 3.0 with attribution requiredu003c/au003e u003ca href="https://stackoverflow.com/legal/content-policy"u003e(content policy)u003c/au003e",
    allowUrls: true
    },
    noCode: true, onDemand: true,
    discardSelector: ".discard-answer"
    ,immediatelyShowMarkdownHelp:true
    });


    }
    });














    draft saved

    draft discarded


















    StackExchange.ready(
    function () {
    StackExchange.openid.initPostLogin('.new-post-login', 'https%3a%2f%2fmath.stackexchange.com%2fquestions%2f3009920%2ffind-the-sum-sum-n-2-infty-frac1n2-1%23new-answer', 'question_page');
    }
    );

    Post as a guest















    Required, but never shown

























    4 Answers
    4






    active

    oldest

    votes








    4 Answers
    4






    active

    oldest

    votes









    active

    oldest

    votes






    active

    oldest

    votes








    up vote
    3
    down vote













    $$frac{1}{n^2-1}=frac{1}{(n-1)(n+1)}=frac{1}{2(n-1)}-frac{1}{2(n+1)}$$



    Taking $frac{1}{2}$ common ,



    $$sum_{n=2}^{infty}frac{1}{n^2-1}=frac12sum_{n=2}^{infty}left(frac{1}{n-1}-frac{1}{n+1}right)$$
    Write the first few terms of the series as :



    $frac{1}{2}( 1 - frac{1}{3} +frac{1}{2} - frac{1}{4} + frac{1}{3} - frac{1}{5} + frac{1}{4} - cdot cdot cdot$



    You can see that except $1$ and $frac{1}{2}$ every terms get cancelled out and the $n$-th term tends to zero. So the result inside the bracket is $frac{3}{2}$ . But there is a $frac{1}{2}$ outside the bracket which needs to be multiplied. So the answer is $frac{3}{4}$.






    share|cite|improve this answer





















    • Yes, this looks ok to me.
      – Jimmy R.
      Nov 23 at 4:27















    up vote
    3
    down vote













    $$frac{1}{n^2-1}=frac{1}{(n-1)(n+1)}=frac{1}{2(n-1)}-frac{1}{2(n+1)}$$



    Taking $frac{1}{2}$ common ,



    $$sum_{n=2}^{infty}frac{1}{n^2-1}=frac12sum_{n=2}^{infty}left(frac{1}{n-1}-frac{1}{n+1}right)$$
    Write the first few terms of the series as :



    $frac{1}{2}( 1 - frac{1}{3} +frac{1}{2} - frac{1}{4} + frac{1}{3} - frac{1}{5} + frac{1}{4} - cdot cdot cdot$



    You can see that except $1$ and $frac{1}{2}$ every terms get cancelled out and the $n$-th term tends to zero. So the result inside the bracket is $frac{3}{2}$ . But there is a $frac{1}{2}$ outside the bracket which needs to be multiplied. So the answer is $frac{3}{4}$.






    share|cite|improve this answer





















    • Yes, this looks ok to me.
      – Jimmy R.
      Nov 23 at 4:27













    up vote
    3
    down vote










    up vote
    3
    down vote









    $$frac{1}{n^2-1}=frac{1}{(n-1)(n+1)}=frac{1}{2(n-1)}-frac{1}{2(n+1)}$$



    Taking $frac{1}{2}$ common ,



    $$sum_{n=2}^{infty}frac{1}{n^2-1}=frac12sum_{n=2}^{infty}left(frac{1}{n-1}-frac{1}{n+1}right)$$
    Write the first few terms of the series as :



    $frac{1}{2}( 1 - frac{1}{3} +frac{1}{2} - frac{1}{4} + frac{1}{3} - frac{1}{5} + frac{1}{4} - cdot cdot cdot$



    You can see that except $1$ and $frac{1}{2}$ every terms get cancelled out and the $n$-th term tends to zero. So the result inside the bracket is $frac{3}{2}$ . But there is a $frac{1}{2}$ outside the bracket which needs to be multiplied. So the answer is $frac{3}{4}$.






    share|cite|improve this answer












    $$frac{1}{n^2-1}=frac{1}{(n-1)(n+1)}=frac{1}{2(n-1)}-frac{1}{2(n+1)}$$



    Taking $frac{1}{2}$ common ,



    $$sum_{n=2}^{infty}frac{1}{n^2-1}=frac12sum_{n=2}^{infty}left(frac{1}{n-1}-frac{1}{n+1}right)$$
    Write the first few terms of the series as :



    $frac{1}{2}( 1 - frac{1}{3} +frac{1}{2} - frac{1}{4} + frac{1}{3} - frac{1}{5} + frac{1}{4} - cdot cdot cdot$



    You can see that except $1$ and $frac{1}{2}$ every terms get cancelled out and the $n$-th term tends to zero. So the result inside the bracket is $frac{3}{2}$ . But there is a $frac{1}{2}$ outside the bracket which needs to be multiplied. So the answer is $frac{3}{4}$.







    share|cite|improve this answer












    share|cite|improve this answer



    share|cite|improve this answer










    answered Nov 23 at 3:47









    Akash Roy

    1




    1












    • Yes, this looks ok to me.
      – Jimmy R.
      Nov 23 at 4:27


















    • Yes, this looks ok to me.
      – Jimmy R.
      Nov 23 at 4:27
















    Yes, this looks ok to me.
    – Jimmy R.
    Nov 23 at 4:27




    Yes, this looks ok to me.
    – Jimmy R.
    Nov 23 at 4:27










    up vote
    2
    down vote













    As you correctly have: $$frac{1}{n^2-1}=frac{1}{(n-1)(n+1)}=frac{1}{2(n-1)}-frac{1}{2(n+1)}$$ Now, observe that $$sum_{n=2}^{infty}frac{1}{n-1}=sum_{n=1}^{infty}frac{1}{n} qquad text{and}qquadsum_{n=2}^{infty}frac{1}{n+1}=sum_{n=3}^{infty}frac{1}{n}$$ Hence
    $$sum_{n=2}^{infty}frac{1}{n^2-1}=frac12sum_{n=2}^{infty}left(frac{1}{n-1}-frac{1}{n+1}right)=frac12left(sum_{n=1}^{infty}frac1n-sum_{n=3}^{infty}frac1nright)=frac12left(frac11+frac12right)=frac34$$






    share|cite|improve this answer



















    • 5




      If you consider partial sums, you can avoid writing out the divergent harmonic series which makes me quite comfortable :)
      – Szeto
      Nov 23 at 3:35






    • 3




      @Szeto: +1 to that. I can prove many things if I'm allowed to use expressions like $sum_{n=1}^{infty}frac{1}{n}$....
      – mjqxxxx
      Nov 23 at 3:39










    • Is my answer okay please check once
      – Akash Roy
      Nov 23 at 3:56















    up vote
    2
    down vote













    As you correctly have: $$frac{1}{n^2-1}=frac{1}{(n-1)(n+1)}=frac{1}{2(n-1)}-frac{1}{2(n+1)}$$ Now, observe that $$sum_{n=2}^{infty}frac{1}{n-1}=sum_{n=1}^{infty}frac{1}{n} qquad text{and}qquadsum_{n=2}^{infty}frac{1}{n+1}=sum_{n=3}^{infty}frac{1}{n}$$ Hence
    $$sum_{n=2}^{infty}frac{1}{n^2-1}=frac12sum_{n=2}^{infty}left(frac{1}{n-1}-frac{1}{n+1}right)=frac12left(sum_{n=1}^{infty}frac1n-sum_{n=3}^{infty}frac1nright)=frac12left(frac11+frac12right)=frac34$$






    share|cite|improve this answer



















    • 5




      If you consider partial sums, you can avoid writing out the divergent harmonic series which makes me quite comfortable :)
      – Szeto
      Nov 23 at 3:35






    • 3




      @Szeto: +1 to that. I can prove many things if I'm allowed to use expressions like $sum_{n=1}^{infty}frac{1}{n}$....
      – mjqxxxx
      Nov 23 at 3:39










    • Is my answer okay please check once
      – Akash Roy
      Nov 23 at 3:56













    up vote
    2
    down vote










    up vote
    2
    down vote









    As you correctly have: $$frac{1}{n^2-1}=frac{1}{(n-1)(n+1)}=frac{1}{2(n-1)}-frac{1}{2(n+1)}$$ Now, observe that $$sum_{n=2}^{infty}frac{1}{n-1}=sum_{n=1}^{infty}frac{1}{n} qquad text{and}qquadsum_{n=2}^{infty}frac{1}{n+1}=sum_{n=3}^{infty}frac{1}{n}$$ Hence
    $$sum_{n=2}^{infty}frac{1}{n^2-1}=frac12sum_{n=2}^{infty}left(frac{1}{n-1}-frac{1}{n+1}right)=frac12left(sum_{n=1}^{infty}frac1n-sum_{n=3}^{infty}frac1nright)=frac12left(frac11+frac12right)=frac34$$






    share|cite|improve this answer














    As you correctly have: $$frac{1}{n^2-1}=frac{1}{(n-1)(n+1)}=frac{1}{2(n-1)}-frac{1}{2(n+1)}$$ Now, observe that $$sum_{n=2}^{infty}frac{1}{n-1}=sum_{n=1}^{infty}frac{1}{n} qquad text{and}qquadsum_{n=2}^{infty}frac{1}{n+1}=sum_{n=3}^{infty}frac{1}{n}$$ Hence
    $$sum_{n=2}^{infty}frac{1}{n^2-1}=frac12sum_{n=2}^{infty}left(frac{1}{n-1}-frac{1}{n+1}right)=frac12left(sum_{n=1}^{infty}frac1n-sum_{n=3}^{infty}frac1nright)=frac12left(frac11+frac12right)=frac34$$







    share|cite|improve this answer














    share|cite|improve this answer



    share|cite|improve this answer








    edited Nov 23 at 3:12

























    answered Nov 23 at 3:02









    Jimmy R.

    33k42157




    33k42157








    • 5




      If you consider partial sums, you can avoid writing out the divergent harmonic series which makes me quite comfortable :)
      – Szeto
      Nov 23 at 3:35






    • 3




      @Szeto: +1 to that. I can prove many things if I'm allowed to use expressions like $sum_{n=1}^{infty}frac{1}{n}$....
      – mjqxxxx
      Nov 23 at 3:39










    • Is my answer okay please check once
      – Akash Roy
      Nov 23 at 3:56














    • 5




      If you consider partial sums, you can avoid writing out the divergent harmonic series which makes me quite comfortable :)
      – Szeto
      Nov 23 at 3:35






    • 3




      @Szeto: +1 to that. I can prove many things if I'm allowed to use expressions like $sum_{n=1}^{infty}frac{1}{n}$....
      – mjqxxxx
      Nov 23 at 3:39










    • Is my answer okay please check once
      – Akash Roy
      Nov 23 at 3:56








    5




    5




    If you consider partial sums, you can avoid writing out the divergent harmonic series which makes me quite comfortable :)
    – Szeto
    Nov 23 at 3:35




    If you consider partial sums, you can avoid writing out the divergent harmonic series which makes me quite comfortable :)
    – Szeto
    Nov 23 at 3:35




    3




    3




    @Szeto: +1 to that. I can prove many things if I'm allowed to use expressions like $sum_{n=1}^{infty}frac{1}{n}$....
    – mjqxxxx
    Nov 23 at 3:39




    @Szeto: +1 to that. I can prove many things if I'm allowed to use expressions like $sum_{n=1}^{infty}frac{1}{n}$....
    – mjqxxxx
    Nov 23 at 3:39












    Is my answer okay please check once
    – Akash Roy
    Nov 23 at 3:56




    Is my answer okay please check once
    – Akash Roy
    Nov 23 at 3:56










    up vote
    1
    down vote













    More generally,
    if
    $s_m(n)
    =sum_{k=m+1}^n frac{1}{k^2-m^2}
    $

    then,
    if $n > 3m$,



    $begin{array}\
    s_m(n)
    &=sum_{k=m+1}^n frac{1}{k^2-m^2}\
    &=sum_{k=m+1}^n frac1{2m}(frac{1}{k-m}-frac1{k+m})\
    &=frac1{2m}sum_{k=m+1}^n frac{1}{k-m}-frac1{2m}sum_{k=m+1}^nfrac1{k+m}\
    &=frac1{2m}(sum_{k=1}^{2m} frac{1}{k}+sum_{k=2m+1}^{n-m} frac{1}{k})-(frac1{2m}sum_{k=2m+1}^{n-m}frac1{k}+frac1{2m}sum_{k=n-m+1}^{n+m}frac1{k})\
    &=frac1{2m}sum_{k=1}^{2m} frac{1}{k}-frac1{2m}sum_{k=n-m+1}^{n+m}frac1{k}\
    end{array}
    $



    so



    $begin{array}\
    s_m(n)-frac1{2m}sum_{k=1}^{2m} frac{1}{k}
    &=-frac1{2m}sum_{k=n-m+1}^{n+m}frac1{k}\
    text{so that}\
    |s_m(n)-frac1{2m}sum_{k=1}^{2m} frac{1}{k}|
    &=frac1{2m}|sum_{k=n-m+1}^{n+m}frac1{k}|\
    &lefrac1{2m}|sum_{k=n-m+1}^{n+m}frac1{n-m+1}|\
    &=frac1{2m}|frac{2m}{n-m+1}|\
    &=frac{1}{n-m+1}\
    &to 0
    quadtext{ as } n to infty\
    end{array}
    $



    Therefore
    $lim_{n to infty} s_m(n)
    =frac1{2m}sum_{k=1}^{2m} frac{1}{k}
    $
    .



    For $m=1$
    the sum is
    $frac1{2}(frac1{1}+frac1{2})
    =frac34
    $
    .






    share|cite|improve this answer

























      up vote
      1
      down vote













      More generally,
      if
      $s_m(n)
      =sum_{k=m+1}^n frac{1}{k^2-m^2}
      $

      then,
      if $n > 3m$,



      $begin{array}\
      s_m(n)
      &=sum_{k=m+1}^n frac{1}{k^2-m^2}\
      &=sum_{k=m+1}^n frac1{2m}(frac{1}{k-m}-frac1{k+m})\
      &=frac1{2m}sum_{k=m+1}^n frac{1}{k-m}-frac1{2m}sum_{k=m+1}^nfrac1{k+m}\
      &=frac1{2m}(sum_{k=1}^{2m} frac{1}{k}+sum_{k=2m+1}^{n-m} frac{1}{k})-(frac1{2m}sum_{k=2m+1}^{n-m}frac1{k}+frac1{2m}sum_{k=n-m+1}^{n+m}frac1{k})\
      &=frac1{2m}sum_{k=1}^{2m} frac{1}{k}-frac1{2m}sum_{k=n-m+1}^{n+m}frac1{k}\
      end{array}
      $



      so



      $begin{array}\
      s_m(n)-frac1{2m}sum_{k=1}^{2m} frac{1}{k}
      &=-frac1{2m}sum_{k=n-m+1}^{n+m}frac1{k}\
      text{so that}\
      |s_m(n)-frac1{2m}sum_{k=1}^{2m} frac{1}{k}|
      &=frac1{2m}|sum_{k=n-m+1}^{n+m}frac1{k}|\
      &lefrac1{2m}|sum_{k=n-m+1}^{n+m}frac1{n-m+1}|\
      &=frac1{2m}|frac{2m}{n-m+1}|\
      &=frac{1}{n-m+1}\
      &to 0
      quadtext{ as } n to infty\
      end{array}
      $



      Therefore
      $lim_{n to infty} s_m(n)
      =frac1{2m}sum_{k=1}^{2m} frac{1}{k}
      $
      .



      For $m=1$
      the sum is
      $frac1{2}(frac1{1}+frac1{2})
      =frac34
      $
      .






      share|cite|improve this answer























        up vote
        1
        down vote










        up vote
        1
        down vote









        More generally,
        if
        $s_m(n)
        =sum_{k=m+1}^n frac{1}{k^2-m^2}
        $

        then,
        if $n > 3m$,



        $begin{array}\
        s_m(n)
        &=sum_{k=m+1}^n frac{1}{k^2-m^2}\
        &=sum_{k=m+1}^n frac1{2m}(frac{1}{k-m}-frac1{k+m})\
        &=frac1{2m}sum_{k=m+1}^n frac{1}{k-m}-frac1{2m}sum_{k=m+1}^nfrac1{k+m}\
        &=frac1{2m}(sum_{k=1}^{2m} frac{1}{k}+sum_{k=2m+1}^{n-m} frac{1}{k})-(frac1{2m}sum_{k=2m+1}^{n-m}frac1{k}+frac1{2m}sum_{k=n-m+1}^{n+m}frac1{k})\
        &=frac1{2m}sum_{k=1}^{2m} frac{1}{k}-frac1{2m}sum_{k=n-m+1}^{n+m}frac1{k}\
        end{array}
        $



        so



        $begin{array}\
        s_m(n)-frac1{2m}sum_{k=1}^{2m} frac{1}{k}
        &=-frac1{2m}sum_{k=n-m+1}^{n+m}frac1{k}\
        text{so that}\
        |s_m(n)-frac1{2m}sum_{k=1}^{2m} frac{1}{k}|
        &=frac1{2m}|sum_{k=n-m+1}^{n+m}frac1{k}|\
        &lefrac1{2m}|sum_{k=n-m+1}^{n+m}frac1{n-m+1}|\
        &=frac1{2m}|frac{2m}{n-m+1}|\
        &=frac{1}{n-m+1}\
        &to 0
        quadtext{ as } n to infty\
        end{array}
        $



        Therefore
        $lim_{n to infty} s_m(n)
        =frac1{2m}sum_{k=1}^{2m} frac{1}{k}
        $
        .



        For $m=1$
        the sum is
        $frac1{2}(frac1{1}+frac1{2})
        =frac34
        $
        .






        share|cite|improve this answer












        More generally,
        if
        $s_m(n)
        =sum_{k=m+1}^n frac{1}{k^2-m^2}
        $

        then,
        if $n > 3m$,



        $begin{array}\
        s_m(n)
        &=sum_{k=m+1}^n frac{1}{k^2-m^2}\
        &=sum_{k=m+1}^n frac1{2m}(frac{1}{k-m}-frac1{k+m})\
        &=frac1{2m}sum_{k=m+1}^n frac{1}{k-m}-frac1{2m}sum_{k=m+1}^nfrac1{k+m}\
        &=frac1{2m}(sum_{k=1}^{2m} frac{1}{k}+sum_{k=2m+1}^{n-m} frac{1}{k})-(frac1{2m}sum_{k=2m+1}^{n-m}frac1{k}+frac1{2m}sum_{k=n-m+1}^{n+m}frac1{k})\
        &=frac1{2m}sum_{k=1}^{2m} frac{1}{k}-frac1{2m}sum_{k=n-m+1}^{n+m}frac1{k}\
        end{array}
        $



        so



        $begin{array}\
        s_m(n)-frac1{2m}sum_{k=1}^{2m} frac{1}{k}
        &=-frac1{2m}sum_{k=n-m+1}^{n+m}frac1{k}\
        text{so that}\
        |s_m(n)-frac1{2m}sum_{k=1}^{2m} frac{1}{k}|
        &=frac1{2m}|sum_{k=n-m+1}^{n+m}frac1{k}|\
        &lefrac1{2m}|sum_{k=n-m+1}^{n+m}frac1{n-m+1}|\
        &=frac1{2m}|frac{2m}{n-m+1}|\
        &=frac{1}{n-m+1}\
        &to 0
        quadtext{ as } n to infty\
        end{array}
        $



        Therefore
        $lim_{n to infty} s_m(n)
        =frac1{2m}sum_{k=1}^{2m} frac{1}{k}
        $
        .



        For $m=1$
        the sum is
        $frac1{2}(frac1{1}+frac1{2})
        =frac34
        $
        .







        share|cite|improve this answer












        share|cite|improve this answer



        share|cite|improve this answer










        answered Nov 23 at 4:06









        marty cohen

        72.1k547126




        72.1k547126






















            up vote
            0
            down vote













            HINT:



            $$frac{1}{2[(n+2)-1]}-frac{1}{2[n+1]}=0$$






            share|cite|improve this answer





















            • Is my answer okay? Please check
              – Akash Roy
              Nov 23 at 3:56















            up vote
            0
            down vote













            HINT:



            $$frac{1}{2[(n+2)-1]}-frac{1}{2[n+1]}=0$$






            share|cite|improve this answer





















            • Is my answer okay? Please check
              – Akash Roy
              Nov 23 at 3:56













            up vote
            0
            down vote










            up vote
            0
            down vote









            HINT:



            $$frac{1}{2[(n+2)-1]}-frac{1}{2[n+1]}=0$$






            share|cite|improve this answer












            HINT:



            $$frac{1}{2[(n+2)-1]}-frac{1}{2[n+1]}=0$$







            share|cite|improve this answer












            share|cite|improve this answer



            share|cite|improve this answer










            answered Nov 23 at 3:03









            Rhys Hughes

            4,6731327




            4,6731327












            • Is my answer okay? Please check
              – Akash Roy
              Nov 23 at 3:56


















            • Is my answer okay? Please check
              – Akash Roy
              Nov 23 at 3:56
















            Is my answer okay? Please check
            – Akash Roy
            Nov 23 at 3:56




            Is my answer okay? Please check
            – Akash Roy
            Nov 23 at 3:56


















            draft saved

            draft discarded




















































            Thanks for contributing an answer to Mathematics Stack Exchange!


            • Please be sure to answer the question. Provide details and share your research!

            But avoid



            • Asking for help, clarification, or responding to other answers.

            • Making statements based on opinion; back them up with references or personal experience.


            Use MathJax to format equations. MathJax reference.


            To learn more, see our tips on writing great answers.





            Some of your past answers have not been well-received, and you're in danger of being blocked from answering.


            Please pay close attention to the following guidance:


            • Please be sure to answer the question. Provide details and share your research!

            But avoid



            • Asking for help, clarification, or responding to other answers.

            • Making statements based on opinion; back them up with references or personal experience.


            To learn more, see our tips on writing great answers.




            draft saved


            draft discarded














            StackExchange.ready(
            function () {
            StackExchange.openid.initPostLogin('.new-post-login', 'https%3a%2f%2fmath.stackexchange.com%2fquestions%2f3009920%2ffind-the-sum-sum-n-2-infty-frac1n2-1%23new-answer', 'question_page');
            }
            );

            Post as a guest















            Required, but never shown





















































            Required, but never shown














            Required, but never shown












            Required, but never shown







            Required, but never shown

































            Required, but never shown














            Required, but never shown












            Required, but never shown







            Required, but never shown







            Popular posts from this blog

            Quarter-circle Tiles

            build a pushdown automaton that recognizes the reverse language of a given pushdown automaton?

            Mont Emei